subject
Mathematics, 20.06.2021 03:10 Samuelmoreno4496

A customer who intends to purchase an appliance has three coupons, only one of which may be used:
Coupon 1: 15% off the listed price if the listed price is at least $100
Coupon 2: $30 off the listed price if the listed price is at least $200
Coupon 3: 20% off the amount by which the listed price exceeds $200
For which of the following listed prices will coupon I not offer a greater price
reduction than either coupon 2 or coupon 3?
A) $279.95 (B) $399.95 (C) $800.99
(C) $800.99 (D) $739.95 (E) $659.95

ansver
Answers: 2

Another question on Mathematics

question
Mathematics, 21.06.2019 19:30
I'm confused on this could anyone me and show me what i need to do to solve it.
Answers: 1
question
Mathematics, 21.06.2019 20:30
In the diagram of circle o, what is the measure of zabc?
Answers: 2
question
Mathematics, 21.06.2019 21:10
What is the equation of a line passing through (-6,5) and having a slope of 1/3
Answers: 3
question
Mathematics, 21.06.2019 22:10
What is the circumference of a circle with an area of 50.24 cm²?
Answers: 2
You know the right answer?
A customer who intends to purchase an appliance has three coupons, only one of which may be used:
Questions
question
Chemistry, 13.04.2021 07:40
question
Mathematics, 13.04.2021 07:40